Question and Answers Forum

All Questions      Topic List

Algebra Questions

Previous in All Question      Next in All Question      

Previous in Algebra      Next in Algebra      

Question Number 60039 by MJS last updated on 17/May/19

find all solutions for z∈C  z^i =(1/2)−(1/2)i  z^(1−i) =1+i

$$\mathrm{find}\:\mathrm{all}\:\mathrm{solutions}\:\mathrm{for}\:{z}\in\mathbb{C} \\ $$$${z}^{\mathrm{i}} =\frac{\mathrm{1}}{\mathrm{2}}−\frac{\mathrm{1}}{\mathrm{2}}\mathrm{i} \\ $$$${z}^{\mathrm{1}−\mathrm{i}} =\mathrm{1}+\mathrm{i} \\ $$

Commented by maxmathsup by imad last updated on 18/May/19

 we have z^i  =e^(iln(z))   so if z=r e^(iθ)   we get   ln(z) =ln(r)+iθ ⇒z^i  =e^(i{ln(r)+iθ})  =e^(−θ)  e^(iln(r)) =e^(−θ) {cos(ln(r) +isin(ln(r)}  z^i =(1/2) −(1/2)i ⇒ e^(−θ) cos(ln(r))=(1/2) and e^(−θ)  sin(ln(r))=−(1/2) ⇒  e^(−2θ)  {cos^2 (lnr) +sin^2 (ln(r))} =(1/4) +(1/4) =(1/2) ⇒e^(−2θ) =(1/2) ⇒−2θ =−ln(2)  ⇒θ =((ln(2))/2)  also we have    ((sin(lnr))/(cos(lnr))) =−1 ⇒tan(lnr) =−1 ⇒  ln(r) =arctan(−1) =−(π/4) ⇒r =e^(−(π/4))  ⇒z =e^(−(π/4))  e^(i((ln(2))/2))   .

$$\:{we}\:{have}\:{z}^{{i}} \:={e}^{{iln}\left({z}\right)} \:\:{so}\:{if}\:{z}={r}\:{e}^{{i}\theta} \:\:{we}\:{get}\: \\ $$$${ln}\left({z}\right)\:={ln}\left({r}\right)+{i}\theta\:\Rightarrow{z}^{{i}} \:={e}^{{i}\left\{{ln}\left({r}\right)+{i}\theta\right\}} \:={e}^{−\theta} \:{e}^{{iln}\left({r}\right)} ={e}^{−\theta} \left\{{cos}\left({ln}\left({r}\right)\:+{isin}\left({ln}\left({r}\right)\right\}\right.\right. \\ $$$${z}^{{i}} =\frac{\mathrm{1}}{\mathrm{2}}\:−\frac{\mathrm{1}}{\mathrm{2}}{i}\:\Rightarrow\:{e}^{−\theta} {cos}\left({ln}\left({r}\right)\right)=\frac{\mathrm{1}}{\mathrm{2}}\:{and}\:{e}^{−\theta} \:{sin}\left({ln}\left({r}\right)\right)=−\frac{\mathrm{1}}{\mathrm{2}}\:\Rightarrow \\ $$$${e}^{−\mathrm{2}\theta} \:\left\{{cos}^{\mathrm{2}} \left({lnr}\right)\:+{sin}^{\mathrm{2}} \left({ln}\left({r}\right)\right)\right\}\:=\frac{\mathrm{1}}{\mathrm{4}}\:+\frac{\mathrm{1}}{\mathrm{4}}\:=\frac{\mathrm{1}}{\mathrm{2}}\:\Rightarrow{e}^{−\mathrm{2}\theta} =\frac{\mathrm{1}}{\mathrm{2}}\:\Rightarrow−\mathrm{2}\theta\:=−{ln}\left(\mathrm{2}\right) \\ $$$$\Rightarrow\theta\:=\frac{{ln}\left(\mathrm{2}\right)}{\mathrm{2}}\:\:{also}\:{we}\:{have}\:\:\:\:\frac{{sin}\left({lnr}\right)}{{cos}\left({lnr}\right)}\:=−\mathrm{1}\:\Rightarrow{tan}\left({lnr}\right)\:=−\mathrm{1}\:\Rightarrow \\ $$$${ln}\left({r}\right)\:={arctan}\left(−\mathrm{1}\right)\:=−\frac{\pi}{\mathrm{4}}\:\Rightarrow{r}\:={e}^{−\frac{\pi}{\mathrm{4}}} \:\Rightarrow{z}\:={e}^{−\frac{\pi}{\mathrm{4}}} \:{e}^{{i}\frac{{ln}\left(\mathrm{2}\right)}{\mathrm{2}}} \:\:. \\ $$

Commented by maxmathsup by imad last updated on 18/May/19

in my solution i have taken the principal determination of log.

$${in}\:{my}\:{solution}\:{i}\:{have}\:{taken}\:{the}\:{principal}\:{determination}\:{of}\:{log}. \\ $$

Commented by maxmathsup by imad last updated on 19/May/19

all solution for this equation are Z_n =e^(−(π/4))  e^(i(((ln(2))/2) +2nπ))

$${all}\:{solution}\:{for}\:{this}\:{equation}\:{are}\:{Z}_{{n}} ={e}^{−\frac{\pi}{\mathrm{4}}} \:{e}^{{i}\left(\frac{{ln}\left(\mathrm{2}\right)}{\mathrm{2}}\:+\mathrm{2}{n}\pi\right)} \\ $$

Answered by tanmay last updated on 17/May/19

(A+iB)^(P+iQ)   =e^((P+iQ)Log(A+iB))   Log(A+iB)  A=rcosθ    B=rsinθ  Log(re^(iθ) )  =Log_e (re^(i(2kπ+θ))   =log_e r+i(2kπ+θ)  =(1/2)log_e (A^2 +B^2 )+i[2kπ+tan^(−1) ((B/A))]  e^((P+iQ)[(1/2)log(A^2 +B^2 )+i{2kπ+tan^(−1) ((B/A))}])   e^p ×e^([{−Q(2kπ+tan^(−1) ((B/A))}+i(Q/2)log(A^2 +B^2 )])   e^P ×e^(−Q(2kπ+tan^(−1) ((B/A))) ×e^(i×(Q/2)log(A^2 +B^2 ))   e^(P−Q(2kπ+tan^(−1) ((B/A))) ×[cos(((Qlog(A^2 +B^2 ))/2))+isin(((Qlog(A^2 +B^2 ))/2))]  wait sir...  now put P=0  and Q=1  e^(−(2kπ+tan^(−1) ((B/A)))) ×[cos(((log(A^2 +B^2 ))/2))+isin(((log(A^2 +B^2 )/2))]  still[to go...

$$\left({A}+{iB}\right)^{{P}+{iQ}} \\ $$$$={e}^{\left({P}+{iQ}\right){Log}\left({A}+{iB}\right)} \\ $$$${Log}\left({A}+{iB}\right) \\ $$$${A}={rcos}\theta\:\:\:\:{B}={rsin}\theta \\ $$$${Log}\left({re}^{{i}\theta} \right) \\ $$$$={Log}_{{e}} \left({re}^{{i}\left(\mathrm{2}{k}\pi+\theta\right)} \right. \\ $$$$={log}_{{e}} {r}+{i}\left(\mathrm{2}{k}\pi+\theta\right) \\ $$$$=\frac{\mathrm{1}}{\mathrm{2}}{log}_{{e}} \left({A}^{\mathrm{2}} +{B}^{\mathrm{2}} \right)+{i}\left[\mathrm{2}{k}\pi+{tan}^{−\mathrm{1}} \left(\frac{{B}}{{A}}\right)\right] \\ $$$${e}^{\left({P}+{iQ}\right)\left[\frac{\mathrm{1}}{\mathrm{2}}{log}\left({A}^{\mathrm{2}} +{B}^{\mathrm{2}} \right)+{i}\left\{\mathrm{2}{k}\pi+{tan}^{−\mathrm{1}} \left(\frac{{B}}{{A}}\right)\right\}\right]} \\ $$$${e}^{{p}} ×{e}^{\left[\left\{−{Q}\left(\mathrm{2}{k}\pi+{tan}^{−\mathrm{1}} \left(\frac{{B}}{{A}}\right)\right\}+{i}\frac{{Q}}{\mathrm{2}}{log}\left({A}^{\mathrm{2}} +{B}^{\mathrm{2}} \right)\right]\right.} \\ $$$${e}^{{P}} ×{e}^{−{Q}\left(\mathrm{2}{k}\pi+{tan}^{−\mathrm{1}} \left(\frac{{B}}{{A}}\right)\right.} ×{e}^{{i}×\frac{{Q}}{\mathrm{2}}{log}\left({A}^{\mathrm{2}} +{B}^{\mathrm{2}} \right)} \\ $$$${e}^{{P}−{Q}\left(\mathrm{2}{k}\pi+{tan}^{−\mathrm{1}} \left(\frac{{B}}{{A}}\right)\right.} ×\left[{cos}\left(\frac{{Qlog}\left({A}^{\mathrm{2}} +{B}^{\mathrm{2}} \right)}{\mathrm{2}}\right)+{isin}\left(\frac{{Qlog}\left({A}^{\mathrm{2}} +{B}^{\mathrm{2}} \right)}{\mathrm{2}}\right)\right] \\ $$$${wait}\:{sir}... \\ $$$${now}\:{put}\:{P}=\mathrm{0}\:\:{and}\:{Q}=\mathrm{1} \\ $$$${e}^{−\left(\mathrm{2}{k}\pi+{tan}^{−\mathrm{1}} \left(\frac{{B}}{{A}}\right)\right)} ×\left[{cos}\left(\frac{{log}\left({A}^{\mathrm{2}} +{B}^{\mathrm{2}} \right)}{\mathrm{2}}\right)+{isin}\left(\frac{{log}\left({A}^{\mathrm{2}} +{B}^{\mathrm{2}} \right.}{\mathrm{2}}\right)\right] \\ $$$${still}\left[{to}\:{go}...\right. \\ $$

Commented by MJS last updated on 18/May/19

nice until here. thanks a lot!  I′m really not sure how to solve these, will  look into it soon...

$$\mathrm{nice}\:\mathrm{until}\:\mathrm{here}.\:\mathrm{thanks}\:\mathrm{a}\:\mathrm{lot}! \\ $$$$\mathrm{I}'\mathrm{m}\:\mathrm{really}\:\mathrm{not}\:\mathrm{sure}\:\mathrm{how}\:\mathrm{to}\:\mathrm{solve}\:\mathrm{these},\:\mathrm{will} \\ $$$$\mathrm{look}\:\mathrm{into}\:\mathrm{it}\:\mathrm{soon}... \\ $$

Answered by Smail last updated on 18/May/19

(1)z^i =(1/2)−(1/2)i=(1/(√2))e^(i(π/4)) =2^(−(1/2)) e^(i((π/4)+2kπ))   z=2^(−(1/(2i))) e^((π/4)+2kπ) =2^(i/2) e^((π/4)+2kπ)   =e^((i/2)ln2) ×e^((π/4)+2kπ)   z=e^((π/4)+2kπ) e^(i((ln2)/2)) =e^((π/4)+2kπ) 2^(i/2)   (2)z^(1−i) =1+i=(√2)e^(i((π/4)+2kπ)) =2^(1/2) e^(i((π/4)+2kπ))   z=2^(1/(2(1−i))) e^((i/(1−i))(π/4+2kπ)) =2^((1+i)/4) ×e^(((i−1)/2)((π/4)+2kπ))   =2^(1/4) ×e^((i/4)ln2) ×e^((i/2)((π/4)+2kπ)) ×e^(−(1/2)((π/4)+2kπ))   z=(2)^(1/4) e^(−((π/4+2kπ)/2)) ×e^(i((π/4)+2kπ+((ln2)/4)))

$$\left(\mathrm{1}\right){z}^{{i}} =\frac{\mathrm{1}}{\mathrm{2}}−\frac{\mathrm{1}}{\mathrm{2}}{i}=\frac{\mathrm{1}}{\sqrt{\mathrm{2}}}{e}^{{i}\frac{\pi}{\mathrm{4}}} =\mathrm{2}^{−\frac{\mathrm{1}}{\mathrm{2}}} {e}^{{i}\left(\frac{\pi}{\mathrm{4}}+\mathrm{2}{k}\pi\right)} \\ $$$${z}=\mathrm{2}^{−\frac{\mathrm{1}}{\mathrm{2}{i}}} {e}^{\frac{\pi}{\mathrm{4}}+\mathrm{2}{k}\pi} =\mathrm{2}^{\frac{{i}}{\mathrm{2}}} {e}^{\frac{\pi}{\mathrm{4}}+\mathrm{2}{k}\pi} \\ $$$$={e}^{\frac{{i}}{\mathrm{2}}{ln}\mathrm{2}} ×{e}^{\frac{\pi}{\mathrm{4}}+\mathrm{2}{k}\pi} \\ $$$${z}={e}^{\frac{\pi}{\mathrm{4}}+\mathrm{2}{k}\pi} {e}^{{i}\frac{{ln}\mathrm{2}}{\mathrm{2}}} ={e}^{\frac{\pi}{\mathrm{4}}+\mathrm{2}{k}\pi} \mathrm{2}^{{i}/\mathrm{2}} \\ $$$$\left(\mathrm{2}\right){z}^{\mathrm{1}−{i}} =\mathrm{1}+{i}=\sqrt{\mathrm{2}}{e}^{{i}\left(\frac{\pi}{\mathrm{4}}+\mathrm{2}{k}\pi\right)} =\mathrm{2}^{\frac{\mathrm{1}}{\mathrm{2}}} {e}^{{i}\left(\frac{\pi}{\mathrm{4}}+\mathrm{2}{k}\pi\right)} \\ $$$${z}=\mathrm{2}^{\frac{\mathrm{1}}{\mathrm{2}\left(\mathrm{1}−{i}\right)}} {e}^{\frac{{i}}{\mathrm{1}−{i}}\left(\pi/\mathrm{4}+\mathrm{2}{k}\pi\right)} =\mathrm{2}^{\frac{\mathrm{1}+{i}}{\mathrm{4}}} ×{e}^{\frac{{i}−\mathrm{1}}{\mathrm{2}}\left(\frac{\pi}{\mathrm{4}}+\mathrm{2}{k}\pi\right)} \\ $$$$=\mathrm{2}^{\frac{\mathrm{1}}{\mathrm{4}}} ×{e}^{\frac{{i}}{\mathrm{4}}{ln}\mathrm{2}} ×{e}^{\frac{{i}}{\mathrm{2}}\left(\frac{\pi}{\mathrm{4}}+\mathrm{2}{k}\pi\right)} ×{e}^{−\frac{\mathrm{1}}{\mathrm{2}}\left(\frac{\pi}{\mathrm{4}}+\mathrm{2}{k}\pi\right)} \\ $$$${z}=\sqrt[{\mathrm{4}}]{\mathrm{2}}{e}^{−\frac{\pi/\mathrm{4}+\mathrm{2}{k}\pi}{\mathrm{2}}} ×{e}^{{i}\left(\frac{\pi}{\mathrm{4}}+\mathrm{2}{k}\pi+\frac{{ln}\mathrm{2}}{\mathrm{4}}\right)} \\ $$

Commented by MJS last updated on 18/May/19

thank you so far. but which values of k are  appropriate?

$$\mathrm{thank}\:\mathrm{you}\:\mathrm{so}\:\mathrm{far}.\:\mathrm{but}\:\mathrm{which}\:\mathrm{values}\:\mathrm{of}\:{k}\:\mathrm{are} \\ $$$$\mathrm{appropriate}? \\ $$

Terms of Service

Privacy Policy

Contact: info@tinkutara.com